LSAT and Law School Admissions Forum

Get expert LSAT preparation and law school admissions advice from PowerScore Test Preparation.

User avatar
 Dave Killoran
PowerScore Staff
  • PowerScore Staff
  • Posts: 5848
  • Joined: Mar 25, 2011
|
#43577
Complete Question Explanation
(The complete setup for this game can be found here: lsat/viewtopic.php?t=8478)

The correct answer choice is (B)

If F is hired with X, then either Template #2 or #3 applies.

Answer choice (A) is incorrect because G could be in S (Template #3).

Answer choice (B) is the correct answer choice. H must be in S.

Answer choice (C) is incorrect because I could be in S (Template #2).

Answer choice (D) is incorrect because W could be in M (Template #3).

Answer choice (E) is incorrect because W could be in P (either template).

Get the most out of your LSAT Prep Plus subscription.

Analyze and track your performance with our Testing and Analytics Package.